Themenbereiche Themenbereiche Profile Hilfe/Anleitungen Help    
Recent Posts Last 1|3|7 Days Suche Suche Tree Tree View  

noch eine Abzählbarkeitsfrage

ZahlReich - Mathematik Hausaufgabenhilfe » Universitäts-Niveau » Zahlentheorie » noch eine Abzählbarkeitsfrage « Zurück Vor »

Das Archiv für dieses Kapitel findest Du hier.

Autor Beitrag
Seitenanfangvoriger Beitragnächster BeitragSeitenende Link zu diesem Beitrag

Gingeralien (Gingeralien)
Suche alle Beiträge dieser Person in dieser Hauptrubrik
Neues Mitglied
Benutzername: Gingeralien

Nummer des Beitrags: 5
Registriert: 04-2003
Veröffentlicht am Donnerstag, den 28. Oktober, 2004 - 17:18:   Beitrag drucken

Es gibt einen Beweis, mit dem man zeigen kann, daß die Menge der reellen Zahlen im Intervall [0,1] nicht abzählbar ist: Dabei umgibt man die i-ten Elemente jeweils mit einem Intervall der Länger 10^i, i=1,2,3...Während die Vereinigung der Intervalle eine vollständige Überabdeckung des Intervalls [0,1] liefert, ist die Summe der Intervalllängen aber <1 (nämlich 1/9, wenn man es mit der geom. Reihe ausrechnet).
->Widerspruch

Meine Frage: Warum kann man das nicht auf die rationalen Zahlen übertragen? Natürlich sind sie abzählbar, aber sie würden doch mit der Vereinigung der obigen Intervalle ebenfalls abgedeckt werden...
Seitenanfangvoriger Beitragnächster BeitragSeitenende Link zu diesem Beitrag

Ingo (Ingo)
Suche alle Beiträge dieser Person in dieser Hauptrubrik
Moderator
Benutzername: Ingo

Nummer des Beitrags: 1025
Registriert: 08-1999
Veröffentlicht am Freitag, den 29. Oktober, 2004 - 13:33:   Beitrag drucken

Kann es sein, dass Du den Beweis etwas zu stark umrissen hast? Mir will nämlich nicht so recht einleuchten, wieso das eine Überdeckung sein soll.
Wenn die 10i als Intervalllänge stimmen, dann hast Du sicher keine Summe von 1 und wenn es 10-i heissen soll, dann wähle ich einfach xi=(1/2)+10-i und erreiche die Hälfte der zu betrachtenden Menge [0,1] nicht einmal.

Wie dem auch sei: Wenn Du das Verfahren auf die Rationalen Zahlen überträgst, wird vermutlich mindestens eine Zahl nicht erwischt. Genauers kann ich erst sagen, wenn ich weiss, was in dem Beweis mit i-ten Element gemeint ist. Denn wie gesagt: eine beliebige Zahl kann es nicht sein.
Seitenanfangvoriger Beitragnächster BeitragSeitenende Link zu diesem Beitrag

Sotux (Sotux)
Suche alle Beiträge dieser Person in dieser Hauptrubrik
Erfahrenes Mitglied
Benutzername: Sotux

Nummer des Beitrags: 448
Registriert: 04-2003
Veröffentlicht am Samstag, den 30. Oktober, 2004 - 11:33:   Beitrag drucken

Hi,

der Beweis setzt doch voraus, dass man mit dem Abzählen alle reellen Zahlen in [0,1] erwischt, und dann muss die Vereinigung aller Intervalle schon deshalb das ganze Intervall [0,1] enthalten. Für Q würde das natürlich nicht gelten !
Seitenanfangvoriger Beitragnächster BeitragSeitenende Link zu diesem Beitrag

Zaph (Zaph)
Suche alle Beiträge dieser Person in dieser Hauptrubrik
Senior Mitglied
Benutzername: Zaph

Nummer des Beitrags: 1721
Registriert: 07-2000
Veröffentlicht am Samstag, den 30. Oktober, 2004 - 16:03:   Beitrag drucken

Du meintest natürlich die Radien 10^-i. Dann ist die Summe der Intervalllängen gleich
1/(1- 1/10) - 1 = 1/9.

In der Tat bleibt, wenn du als Mittelpunkte die rationalen Zahlen nimmst, mehr als 9/10 des Intervalls unüberdeckt. Allerdings wirst du dann kein unüberdecktes Intervall finden (denn das enthält ja immer eine rationale Zahl!)

Beitrag verfassen
Das Senden ist in diesem Themengebiet nicht unterstützt. Kontaktieren Sie den Diskussions-Moderator für weitere Informationen.

ad

Administration Administration Abmelden Abmelden   Previous Page Previous Page Next Page Next Page